Showing a sequence converges to 0

  • Thread starter trap101
  • Start date
  • Tags
    Sequence
In summary, the conversation discusses a problem involving showing that 2^n/n! converges to 0 by proving the inequality 2^n/n! <= 4/n. The solution involves using induction and factoring out 2^k/k! from the inequality. It is also noted that the base case must be verified for n = 1 and n = 2 in order for the inductive step to hold.
  • #1
trap101
342
0
Show that 2^n/n! Converges to 0 by showing that 2^n/n! <= 4/n
My problem is in showing 2^n/n! <= 4/n

I know 4/n goez to 0, but how to get the inequality to be true? Doing some rough work I noticed that it "appears" to be true for n>=3. Would induction be the way to go about this step? Because nothing else is I am thinking of is coming close to a full thought.
 
Physics news on Phys.org
  • #2
Yes, try induction.
 
  • #3
I am actually having a little problem with the induction.

Assuming all the base case steps are done and we are assuming 2^k/k! <= 4/(k) is true:

2^(k+1)/(k+1)! <= 4/(k+1) [trying to prove]

I am trying to fiddle with it, but nothing much has come up as constructive.

Perhaps: 4/(k+1) = 2 [2/(k+1)] > 2/(K+1), But I am still left trying to show 2/(k+1) > 2^(k+1)/(k+1)!
 
  • #4
Tip: factor out 2^k/k! out of 2^(k+1)/(k+1)! and use the inductive hypothesis.
 
  • #5
I tried that before and got

(2^k/k!)(2/(k+1)) <= (4/k)(2/(k+1))

Now by inductive hypothesis I know already that (2^k/k!) <= (4/k), and just multiplying both sides of that doesn't change that, but I feel I still need to have 4/(k+1) to appear in the inequality somewhere.
 
  • #6
You have this

(2^k/k!)(2/(k+1)) <= (4/k)(2/(k+1))

so it suffices to prove that

(4/k)(2/(k+1)) <= 4/(k+1)
 
  • #7
trap101 said:
I tried that before and got

(2^k/k!)(2/(k+1)) <= (4/k)(2/(k+1))

Now by inductive hypothesis I know already that (2^k/k!) <= (4/k), and just multiplying both sides of that doesn't change that, but I feel I still need to have 4/(k+1) to appear in the inequality somewhere.

By the magic of commutativity of multiplication,
[tex]
\frac{2}{k + 1} \frac{4}{k} = \frac{2}{k} \frac{4}{k + 1}.
[/tex] What condition must 2/k satisfy if this is to be less than or equal to 4/(k + 1)?
 
  • #8
I think I may be showing my inexperience here, but I'm struggling to see how the other posts are helping (although I'm sure they will be doing just that!).

But personally, my thoughts on it are:

trap101 said:
Assuming all the base case steps are done and we are assuming 2^k/k! <= 4/(k) is true:

2^(k+1)/(k+1)! <= 4/(k+1) [trying to prove]
...you have started by stating what you need to prove. Surely you should be stating what you know to be true (given the assumption) and then deducing what you need to prove.
 
  • #9
disregardthat said:
You have this

(2^k/k!)(2/(k+1)) <= (4/k)(2/(k+1))

so it suffices to prove that

(4/k)(2/(k+1)) <= 4/(k+1)



Bloody hell, that's what I was missing to do, I've been trying to polish up on these processes for the new year. So now there is no factorial which makes it a lot easier. Of course it would be induction again on this piece correct, but no factorial should make it more straight forward. If I have an issue I will ask for help. Thanks
 
  • #10
pasmith said:
By the magic of commutativity of multiplication,
[tex]
\frac{2}{k + 1} \frac{4}{k} = \frac{2}{k} \frac{4}{k + 1}.
[/tex] What condition must 2/k satisfy if this is to be less than or equal to 4/(k + 1)?



Ok so I thought I had it figured out, but apparently not. So moving along we've established that it suffices to show that (4/k)(2/(k+1)) <= 4/(k+1) and I will be done.

So going with the hint from pasmith I found that if 2 <= K then this inequality will hold. But I feel I am still missing something in terms of induction. Don't I have to find a middle value of some sort between (4/k)(2/(k+1)) and 4/(k+1) that I could use to show that it (4/k)(2/(k+1)) is less than 4/(k+1) ?
 
  • #11
trap101 said:
Ok so I thought I had it figured out, but apparently not. So moving along we've established that it suffices to show that (4/k)(2/(k+1)) <= 4/(k+1) and I will be done.

So going with the hint from pasmith I found that if 2 <= K then this inequality will hold. But I feel I am still missing something in terms of induction. Don't I have to find a middle value of some sort between (4/k)(2/(k+1)) and 4/(k+1) that I could use to show that it (4/k)(2/(k+1)) is less than 4/(k+1) ?

You want to show that [itex]0 < x < y[/itex]. You know that [itex]x = ay[/itex] where [itex]0 < a < 1[/itex]. But that's sufficient because, by an axiom of the rational number system, if [itex]a < 1[/itex] then multiplying both sides by the positive number [itex]y[/itex] will preserve the inequality: [itex]ay < y[/itex].

Thus it is enough to note that [itex]2/k \leq 1[/itex] when [itex]k \geq 2[/itex], which [itex]k[/itex] is because [itex]k[/itex] is a positive integer and you aren't using this argument to establish the base case [itex]k = 1[/itex]; you will do that by direct substitution.
 
  • #12
Ok so if I may summarize to make sure I got the thinking right.

So at our inductive we assumed that 2^k/k! <= 4/k.

decomposing 2^(k+1)/(k+1)! = (2^k/k!)(2/(k+1))

so this provides me with the hint that I multiply both sides of 2^k/k! <= 4/k by (2/(k+1))

now since by assumption we know 2^k/k! <= 4/k is true, multiplying the same value on both sides does not affect it, which leaves me with showing that if (4/k)(2/(k+1)) <= 4/(k+1) is true, then I have established the result.

Now after some work (4/k)(2/(k+1)) <= 4/(k+1) is true as long as K >= 2 because by the commutativity of multiplication: (4/k)(2/(k+1)) = (2/k)(4/(k+1)) and as long as 2/k <= 1 this will be the case. Since we are showing that this holds for all positive integers and this statement is true for k >= 2, then we have established that (4/k)(2/(k+1)) <= 4/(k+1) and the result follows.
 
  • #13
Note here that you are required to verify this for n = 1 AND n = 2 for the inductive step to hold. Part of the assumption must be that k >= 2, which means that the case n = 2 must be part of the base case.
 
  • #14
Got it. Thanks all for the help.
 
  • #15
pasmith said:
You want to show that [itex]0 < x < y[/itex]. You know that [itex]x = ay[/itex] where [itex]0 < a < 1[/itex]. But that's sufficient because, by an axiom of the rational number system, if [itex]a < 1[/itex] then multiplying both sides by the positive number [itex]y[/itex] will preserve the inequality: [itex]ay < y[/itex].
Thanks. Good to know. This is exactly what I was trying to hint at in post #8. Still not fully understanding your post #7 though!

disregardthat said:
Note here that you are required to verify this for n = 1 AND n = 2 for the inductive step to hold. Part of the assumption must be that k >= 2, which means that the case n = 2 must be part of the base case.
No, not getting this at all. Why must n=2 be part of the base assumption?

The statement is true when n=1 (true for n=0, in fact - not sure I would reach out for n<0 as I'm not too clear on negative factorials just yet - I'm only in my 40s and not reached that part of my study just yet!), and it can be proved for all integer k+1 if we assume true for any integer k. Therefore the direct proof of case k=2 isn't necessary at all. Of course, proving that the case n=2 is true could be put forward as part of your base assumption, as, well, yes, it's obviously true! But not a necessary assumption.

If this isn't the case then either I need to go back and study maths, or have a long lie down. :confused:
 
Last edited:
  • #16
You have proven that the statement, call it [itex]P(n)[/itex], is such that *) [itex]P(k) \Rightarrow P(k+1)[/itex] for [itex]k \geq 2[/itex]. If you initially show [itex]P(1)[/itex], and then the chain of implications [itex]P(2) \Rightarrow P(3) \Rightarrow P(4) \Rightarrow ...[/itex] which is what *) basically says, then you can never make the leap from [itex]P(1)[/itex] to [itex]P(2)[/itex]. You need to know [itex]P(2)[/itex] in order to conclude with [itex]P(3)[/itex], and then [itex]P(4)[/itex], and so on...
 

1. What does it mean for a sequence to converge to 0?

When a sequence converges to 0, it means that as the terms of the sequence get larger, they eventually get closer and closer to the value of 0. In other words, the sequence approaches 0 as its limit.

2. How is convergence to 0 different from divergence?

Convergence to 0 means that the terms of the sequence get closer to 0 as the sequence progresses, while divergence means that the terms of the sequence do not approach a specific value and may increase or decrease without bound.

3. What are the necessary conditions for a sequence to converge to 0?

For a sequence to converge to 0, the terms of the sequence must get closer and closer to 0 as the sequence progresses. In addition, the sequence must also be bounded, meaning that there is a maximum and minimum value for the terms of the sequence.

4. How can you prove that a sequence converges to 0?

To prove that a sequence converges to 0, you can use the definition of convergence, which states that for any positive real number ε, there exists a positive integer N such that for all n ≥ N, the absolute value of the difference between the term of the sequence and 0 is less than ε. Essentially, this means that as the terms of the sequence get larger, they eventually get closer and closer to 0 within a certain tolerance.

5. Can a sequence converge to 0 without having a finite limit?

Yes, a sequence can converge to 0 without having a finite limit. This is known as oscillation, where the terms of the sequence continuously alternate between positive and negative values while getting closer to 0. In this case, the sequence does not have a single limit, but it still converges to 0.

Similar threads

  • Calculus and Beyond Homework Help
Replies
1
Views
258
  • Calculus and Beyond Homework Help
Replies
4
Views
309
  • Calculus and Beyond Homework Help
Replies
34
Views
2K
  • Calculus and Beyond Homework Help
Replies
5
Views
991
  • Calculus and Beyond Homework Help
Replies
3
Views
416
  • Calculus and Beyond Homework Help
Replies
6
Views
390
  • Calculus and Beyond Homework Help
Replies
4
Views
897
  • Calculus and Beyond Homework Help
Replies
9
Views
1K
  • Calculus and Beyond Homework Help
Replies
3
Views
843
  • Calculus and Beyond Homework Help
Replies
7
Views
926
Back
Top